Physics 2 Final "Review"

Pataasin ang iyong marka sa homework at exams ngayon gamit ang Quizwiz!

A capacitor consists of a set of two parallel plates of area A separated by a distance d. This capacitor is connected to a battery that maintains a constant potential difference between the plates. A slab of a dielectric material is inserted in the region between the plates so as to completely fill it. What changes would you observe? Select one: a. Both the charge in the capacitor and its capacitance would change. Correct b. Nothing would change. c. Only the capacitance would change. d. Only the charge in the capacitor would change. Feedback The correct answer is: Both the charge in the capacitor and its capacitance would change. Question 2 Correct Mark 1.00 out of 1.00 Not flaggedFlag question Question text Electrons move in an electrical circuit Select one: a. by interacting with an established electric field. Correct b. by colliding with molecules. c. by being bumped by other electrons. d. none of these e. because the wires are so thin. Feedback The correct answer is: by interacting with an established electric field. Question 3 Correct Mark 1.00 out of 1.00 FlaggedRemove flag Question text Three identical resistors R are connected in series to a voltage source V. The total current, I, suppllied by the voltage source is equal to . . . Select one: a. mc022-4.jpg b. mc022-1.jpg c. mc022-2.jpg Correct d. mc022-3.jpg Feedback The correct answer is: mc022-2.jpg Question 4 Correct Mark 1.00 out of 1.00 Not flaggedFlag question Question text Three identical resistors R are connected in series to a voltage source V. The current, I, through each resistor is equal to . . . Select one: a. mc021-1.jpg b. mc021-2.jpg Correct c. mc021-4.jpg d. mc021-3.jpg Feedback The correct answer is: mc021-2.jpg Question 5 Correct Mark 1.00 out of 1.00 Not flaggedFlag question Question text Three identical resistors are connected in series to a 12-V battery. What is the voltage across any one of the resistors? Select one: a. 36 V b. zero c. 4 V Correct d. 12 V Feedback The correct answer is: 4 V Question 6 Correct Mark 1.00 out of 1.00 Not flaggedFlag question Question text Which of the following is the correct unit for capacitance of a capacitor? Select one: a. Nm/C2 b. J/C c. C/J d. C/J2 e. C2/J Correct Feedback The correct answer is: C2/J Question 7 Correct Mark 1.00 out of 1.00 Not flaggedFlag question Question text The length of a certain wire is doubled and at the same time its radius is increased by a factor of 4. What is the change in the resistance of this wire? Select one: a. It increases by a factor of 8. b. It stays the same. c. It is reduced by a factor of 8. Correct d. It increases by a factor of 4. e. It is reduced by a factor of 4. Feedback The correct answer is: It is reduced by a factor of 8. Question 8 Correct Mark 1.00 out of 1.00 Not flaggedFlag question Question text Three identical resistors R are connected in parallel to a voltage source V. The current, I, through each resistor is equal to . . . Select one: a. mc024-2.jpg b. mc024-1.jpg Correct c. mc024-3.jpg d. mc024-4.jpg Feedback The correct answer is: mc024-1.jpg Question 9 Correct Mark 1.00 out of 1.00 Not flaggedFlag question Question text Stretch a copper wire so that it is thinner and the resistance between its ends Select one: a. remains unchanged. b. increases. Correct c. approaches infinity d. decreases. Feedback The correct answer is: increases. Question 10 Correct Mark 1.00 out of 1.00 Not flaggedFlag question Question text In the circuit shown below, which bulb, when burned out, will cause all other bulbs to stop glowing? mc002-1.jpg Select one: a. F b. C c. A Correct d. B Feedback The correct answer is: A Question 11 Correct Mark 1.00 out of 1.00 Not flaggedFlag question Question text As more resistors are added in parallel to a constant voltage source, the power supplied by the source Select one: a. decreases. b. increases. Correct c. increases for a time and then starts to decrease. d. does not change. Feedback The correct answer is: increases. Question 12 Correct Mark 1.00 out of 1.00 Not flaggedFlag question Question text Four unequal resistors are connected in a series circuit. Which one of the following statements is correct about this circuit? Select one: a. The total resistance is equal to average of the four resistors. b. The total resistance is equal to any one of the resistors. c. The total resistance is less than the largest resistor. d. The total resistance is less than the smallest resistor. e. The total resistance is more than the largest resistor. Correct Feedback The correct answer is: The total resistance is more than the largest resistor. Question 13 Correct Mark 1.00 out of 1.00 Not flaggedFlag question Question text Which of the following circuits can be used to test whether or not a wire of a laminated heating element is cut internally? Select one: a. mc001-2.jpg Correct b. mc001-3.jpg c. mc001-1.jpg d. mc001-4.jpg Feedback The correct answer is: mc001-2.jpg Question 14 Correct Mark 1.00 out of 1.00 Not flaggedFlag question Question text Kirchhoff's loop rule is a statement of Select one: a. the law of conservation of charge. b. Newton's second law. c. the law of conservation of energy. Correct d. the law of conservation of momentum. e. the law of conservation of angular momentum. Feedback The correct answer is: the law of conservation of energy. Question 15 Correct Mark 1.00 out of 1.00 Not flaggedFlag question Question text In an electric circuit, the safety fuse is connected to the circuit in Select one: a. either series or parallel. b. series. Correct c. parallel. d. a counter-clockwise direction Feedback The correct answer is: series. Question 16 Correct Mark 1.00 out of 1.00 Not flaggedFlag question Question text Three identical resistors are connected in parallel to a battery. If the current of 12 A flows from the battery, how much current flows through any one of the resistors? Select one: a. zero b. 36 A c. 4 A Correct d. 12 A Feedback The correct answer is: 4 A Question 17 Correct Mark 1.00 out of 1.00 Not flaggedFlag question Question text When resistors are connected in series, Select one: a. More than one of the given answers is true. b. the current flowing in each is the same. Correct c. the potential difference across each is the same. d. the same power is dissipated in each one. Feedback The correct answer is: the current flowing in each is the same. Question 18 Correct Mark 1.00 out of 1.00 Not flaggedFlag question Question text As more resistors are added in series to a constant voltage source, the power supplied by the source Select one: a. does not change. b. increases. c. decreases. Correct d. increases for a time and then starts to decrease. Feedback The correct answer is: decreases. Question 19 Correct Mark 1.00 out of 1.00 Not flaggedFlag question Question text A capacitor consists of a set of two parallel plates of area A separated by a distance d. This capacitor is connected to a battery that maintains a constant potential difference between the plates. If the separation between the plates is doubled, the magnitude of the charge on the plates will Select one: a. be cut in half. Correct b. be cut in fourth. c. double. d. not change. e. quadruple. Feedback The correct answer is: be cut in half. Question 20 Correct Mark 1.00 out of 1.00 Not flaggedFlag question Question text A dielectric material such as paper is placed between the plates of a capacitor holding a fixed charge. What happens to the electric field between the plates? Select one: a. reduces to zero b. no change c. becomes stronger d. becomes weaker Correct Feedback The correct answer is: becomes weaker Question 21 Correct Mark 1.00 out of 1.00 Not flaggedFlag question Question text A capacitor consists of a set of two parallel plates of area A separated by a distance d. This capacitor is connected to a battery and charged until its plates carry charges +Q and -Q. If the separation between the plates is doubled, the potential difference between the plates will Select one: a. quadruple. b. not change. c. double. Correct d. be cut in fourth. e. be cut in half. Feedback The correct answer is: double. Question 22 Correct Mark 1.00 out of 1.00 Not flaggedFlag question Question text When resistors are connected in parallel, we can be certain that Select one: a. the power dissipated in each is the same. b. the potential difference across each is the same. Correct c. their equivalent resistance is greater than the resistance of any one of the individual resistances. d. the same current flows in each one. Feedback The correct answer is: the potential difference across each is the same. Question 23 Correct Mark 1.00 out of 1.00 Not flaggedFlag question Question text Which of the following will increase the capacitance between the plates of a parallel plate capacitor? Select one: a. Introduce a dielectric material between the plates. Correct b. Increase the potential between the plates. c. Increase the charge on the plates. d. Decrease the potential between the plates. e. none of the above Feedback The correct answer is: Introduce a dielectric material between the plates. Question 24 Correct Mark 1.00 out of 1.00 Not flaggedFlag question Question text A capacitor consists of a set of two parallel plates of area A separated by a distance d. This capacitor is connected to a battery and charged until its plates carry charges +Q and -Q. If the separation between the plates is doubled, the electrical energy stored in the capacitor will Select one: a. double. Correct b. be cut in fourth. c. not change. d. quadruple. e. be cut in half. Feedback The correct answer is: double. Question 25 Correct Mark 1.00 out of 1.00 Not flaggedFlag question Question text Which one of the following represents the correct unit for electric power? Select one: a. J/s b. All of the above answers are correct. Correct c. W d. None of the above answers is correct. e. Nm/s Feedback The correct answer is: All of the above answers are correct. Question 26 Correct Mark 1.00 out of 1.00 Not flaggedFlag question Question text Heat a copper wire and its electric resistance Select one: a. remains unchanged. b. decreases. c. flows towards the heat source d. increases. Correct Feedback The correct answer is: increases. Question 27 Correct Mark 1.00 out of 1.00 Not flaggedFlag question Question text A circuit breaker often serves the same purpose as a Select one: a. None of the above choices are correct. b. fuse. Correct c. capacitor. d. battery e. All of the above choices are correct. Feedback The correct answer is: fuse. Question 28 Correct Mark 1.00 out of 1.00 Not flaggedFlag question Question text The lamps in a string of Christmas tree lights are connected in parallel. What happens if one lamp burns out? (Assume negligible resistance in the wires leading to the lamps.) Select one: a. The other lamps get brighter, but some get brighter than others. b. The other lamps get brighter equally. c. The brightness of the lamps will not change appreciably. Correct d. The other lamps get dimmer, but some get dimmer than others. e. The other lamps get dimmer equally. Feedback The correct answer is: The brightness of the lamps will not change appreciably. Question 29 Correct Mark 1.00 out of 1.00 Not flaggedFlag question Question text A circuit is powered with a battery. Charge flows Select one: a. out of the battery and into the circuit. b. through both the battery and the rest of the circuit. Correct c. none of these d. from the negative battery terminal to the positive terminal. e. only after a couple seconds pass. Feedback The correct answer is: through both the battery and the rest of the circuit. Question 30 Correct Mark 1.00 out of 1.00 Not flaggedFlag question Question text When two or more resistors are connected in parallel to a battery, Select one: a. the total current flowing from the battery equals the sum of the currents flowing through each resistor. b. the voltage across each resistor is the same. c. all of the given answers Correct d. the equivalent resistance of the combination is less than the resistance of any one of the resistors. Feedback The correct answer is: all of the given answers Question 31 Correct Mark 1.00 out of 1.00 Not flaggedFlag question Question text A capacitor consists of a set of two parallel plates of area A separated by a distance d. This capacitor is connected to a battery that maintains a constant potential difference between the plates. If the separation between the plates is doubled, the magnitude of the electrical energy stored on the capacitor will Select one: a. not change. b. be cut in half. Correct c. quadruple. d. be cut in fourth. e. double. Feedback The correct answer is: be cut in half. Question 32 Correct Mark 1.00 out of 1.00 Not flaggedFlag question Question text Which statement is correct? Select one: a. Charge flows in a closed circuit. Correct b. Voltage flows through an open or a closed circuit. c. Resistance flows though an open circuit. d. Current is the primary cause of voltage. Feedback The correct answer is: Charge flows in a closed circuit. Question 33 Correct Mark 1.00 out of 1.00 Not flaggedFlag question Question text Three identical resistors are connected in parallel to a 12-V battery. What is the voltage of any one of the resistors? Select one: a. 4 V b. zero c. 12 V Correct d. 36 V Feedback The correct answer is: 12 V Question 34 Correct Mark 1.00 out of 1.00 Not flaggedFlag question Question text Three identical resistors are connected in series to a battery. If the current of 12 A flows from the battery, how much current flows through any one of the resistors? Select one: a. zero b. 12 A Correct c. 4 A d. 36 A Feedback The correct answer is: 12 A Question 35 Correct Mark 1.00 out of 1.00 Not flaggedFlag question Question text Which of the following expression(s) represents the electrical energy stored by a capacitor? Select one: a. CV2/2 b. All of the expressions are correct. Correct c. QV/2 d. C2/2C e. None of the expressions is correct. Feedback The correct answer is: All of the expressions are correct. Question 36 Correct Mark 1.00 out of 1.00 Not flaggedFlag question Question text Four unequal resistors are connected in a parallel circuit. Which one of the following statements is correct about this circuit? Select one: a. None of the other answers is correct. b. The total resistance is equal to the average of the resistance of all the resistors. c. The total resistance is less than the smallest resistor. Correct d. The total resistance is equal to average of the four resistors. e. The total resistance is more than the largest resistor. Feedback The correct answer is: The total resistance is less than the smallest resistor. Question 37 Correct Mark 1.00 out of 1.00 Not flaggedFlag question Question text An ampere is a unit of electrical Select one: a. all of these b. current. Correct c. pressure. d. none of these e. resistance. Feedback The correct answer is: current. Question 38 Correct Mark 1.00 out of 1.00 Not flaggedFlag question Question text When the current through a resistor is increased by a factor of 4, the power dissipated by it Select one: a. decreases by a factor of 16. b. increases by a factor of 16. Correct c. remain unchanged. d. increases by a factor of 4. e. decreases by a factor of 4. Feedback The correct answer is: increases by a factor of 16. Question 39 Correct Mark 1.00 out of 1.00 Not flaggedFlag question Question text Kirchhoff's junction rule is a statement of Select one: a. the law of conservation of charge. Correct b. Newton's second law. c. the law of conservation of angular momentum. d. the law of conservation of energy. e. the law of conservation of momentum. Feedback The correct answer is: the law of conservation of charge. Question 40 Correct Mark 1.00 out of 1.00 FlaggedRemove flag Question text Three identical resistors R are connected in parallel to a voltage source V. The total current, I, suppllied by the voltage source is equal to . . . Select one: a. mc023-2.jpg b. mc023-1.jpg c. mc023-4.jpg d. mc023-3.jpg Correct Feedback The correct answer is: mc023-3.jpg Question 41 Correct Mark 1.00 out of 1.00 Not flaggedFlag question Question text When a dielectric material is introduced between the plates of a parallel plate capacitor the capacitance increases by a factor of 4. What is the dielectric constant of the material introduced between the plates? Select one: a. 1/4 b. None of the other choices is correct. c. 4 Correct d. 0.4 e. 2 Feedback The correct answer is: 4 Question 42 Correct Mark 1.00 out of 1.00 Not flaggedFlag question Question text Which of the following will increase the capacitance of a parallel plate capacitor? Select one: a. a decrease in the plate area and an increase in the plate separation b. an increase in the potential difference between the plates c. a decrease in the potential difference between the plates d. none of the above e. an increase in the plate area and a decrease in the plate separation Correct Feedback The correct answer is: an increase in the plate area and a decrease in the plate separation

Capacitors

A conductor differs from an insulator in that a conductor Select one: a. has faster moving molecules. b. none of these c. has more protons than electrons. d. has more electrons than protons. e. has more energy than an insulator. Feedback The correct answer is: none of these Question 2 Not answered Marked out of 1.00 Not flaggedFlag question Question text A main difference between gravitational and electric forces is that electrical forces Select one: a. obey the inverse-square law. b. act over shorter distances. c. are weaker. d. attract. e. repel or attract. Feedback The correct answer is: repel or attract. Question 3 Not answered Marked out of 1.00 Not flaggedFlag question Question text A negatively charged balloon sticks to a wooden door. However, an uncharged balloon does not stick to a wooden door. What is the nature of the charge on the wooden door? Select one: a. Positively charged b. Electrically neutral c. Conductor d. Lacking protons Feedback When a negatively charged balloon is brought near a wooden door, the negative charge of the balloon pushes the electrons of the wood molecules away from the location of contact. The charged balloon causes the wood molecules to polarize causing attraction between the two objects. The correct answer is: Electrically neutral Question 4 Not answered Marked out of 1.00 Not flaggedFlag question Question text A negatively charged rod is brought near one end of an uncharged metal bar. The end of the metal bar farthest from the charged rod will be charged Select one: a. positive. b. negative. c. neutral. d. none of the given answers Feedback The correct answer is: negative. Question 5 Not answered Marked out of 1.00 Not flaggedFlag question Question text An electron and a proton Select one: a. attract each other. b. neither attract or repel each other. c. attract or repel dependig upon separation distance d. repel each other. Feedback The correct answer is: attract each other. Question 6 Not answered Marked out of 1.00 Not flaggedFlag question Question text An electroscope is charged positively as shown by foil leaves that stand apart. As a negative charge is brought close to the electroscope, the leaves Select one: a. spread apart further. b. emit sparks c. do not move. d. fall closer together. Feedback The correct answer is: fall closer together. Question 7 Not answered Marked out of 1.00 Not flaggedFlag question Question text An negatively charged pith ball is suspended by a nylon fiber. When a neutrally charged rubber rod is brought nearby, without touching it, the pith ball Select one: a. is repelled by the rod. b. is unaffected by the rod c. is first attracted then repelled by the rod d. is attracted to the rod Feedback The correct answer is: is attracted to the rod Question 8 Not answered Marked out of 1.00 Not flaggedFlag question Question text An uncharged pith ball is suspended by a nylon fiber. When a negatively charged rubber rod is brought nearby, without touching it, the pith ball Select one: a. in unaffected. b. becomes charged by induction. c. None of the above choices are correct. d. is repelled by the rod. e. becomes polarized. Feedback The correct answer is: becomes polarized. Question 9 Not answered Marked out of 1.00 Not flaggedFlag question Question text Is it possible for a positive and a negative charge to attract each other? Select one: a. Yes, they will attract if they are close enough. b. Yes, they will attract if one carries a larger charge than the other. c. Yes, they always attract. d. No, they will never attract. Feedback The correct answer is: Yes, they always attract. Question 10 Not answered Marked out of 1.00 Not flaggedFlag question Question text Is it possible for two negative charges to attract each other? Select one: a. Yes, they will attract if they are close enough. b. Yes, they always attract. c. Yes, they will attract if one carries a larger charge than the other. d. No, they will never attract. Feedback The correct answer is: No, they will never attract. Question 11 Not answered Marked out of 1.00 Not flaggedFlag question Question text Materials in which the electrons are bound very loosely to the nuclei and can move about freely within the material are referred to as Select one: a. insulators. b. semiconductors. c. superconductors. d. conductors. Feedback The correct answer is: conductors. Question 12 Not answered Marked out of 1.00 Not flaggedFlag question Question text Particle A has twice the charge of nearby particle B. Compared to the force on Particle A, the force on Particle B is Select one: a. four times as much. b. half as much. c. twice as much. d. None of the above choices are correct. e. the same. Feedback The correct answer is: the same. Question 13 Not answered Marked out of 1.00 Not flaggedFlag question Question text Sphere A carries a net positive charge, and sphere B is neutral. They are placed near each other on an insulated table. Sphere B is briefly touched with a wire that is grounded. Which statement is correct? Select one: a. Sphere B is now positively charged. b. The charge on sphere B cannot be determined without additional information. c. Sphere B is now negatively charged, d. Sphere B remains neutral. Feedback The correct answer is: Sphere B is now negatively charged, Question 14 Not answered Marked out of 1.00 Not flaggedFlag question Question text The distance between two charges mc006-1.jpg and mc006-2.jpg is r, and the force between them is F. What is the force between them if the distance between them is quartered? Select one: a. F/4 b. 2F c. 16F d. F/9 Feedback The magnitude of force between charges qa and qb, separated by a distance r, is proportional to the magnitude of charges and inversely proportional to the square of the distance between them. The correct answer is: 16F Question 15 Not answered Marked out of 1.00 Not flaggedFlag question Question text The electrical force between charges depends only on the the charges Select one: a. None of the above choices are correct. b. magnitude . c. separation distance d. magnitude and separation distance. Feedback The correct answer is: magnitude and separation distance. Question 16 Not answered Marked out of 1.00 Not flaggedFlag question Question text The electrical force between charges is strongest when the charges are Select one: a. far apart. b. close together. c. The electric force is constant everywhere. Feedback The correct answer is: close together. Question 17 Not answered Marked out of 1.00 Not flaggedFlag question Question text The unit of electric charge, the coulomb, is the charge on Select one: a. one electron. b. a quark. c. a neutron. d. a specific number of neutrons. e. a specific large number of electrons. Feedback The correct answer is: a specific large number of electrons. Question 18 Not answered Marked out of 1.00 Not flaggedFlag question Question text Three rods of different materials P, Q, and R, are charged by various methods. When the rods are brought near each other, the rods P and Q repel each other, while the rods P and R attract each other. Which of the following could be the signs of the charges on the rods? Rod P Rod Q Rod R A - + + B - + - C - - + D - - - Select one: a. - + + b. - + - c. - - + d. - - - Feedback If objects are similarly charged, the force between them is repulsive. However, if the objects are oppositely charged, the force between them is attractive. The correct answer is: - - + Question 19 Not answered Marked out of 1.00 Not flaggedFlag question Question text To say that electric charge is conserved is to say that electric charge Select one: a. is sometimes negative. b. will interact with neighboring electric charges. c. may occur in an infinite variety of quantities. d. is a whole number multiple of the charge of one electron. e. can be neither created nor destroyed. Feedback The correct answer is: can be neither created nor destroyed. Question 20 Not answered Marked out of 1.00 Not flaggedFlag question Question text Two charged objects are separated by a distance d. The first charge is larger in magnitude than the second charge. Select one: a. The charges exert forces on each other equal in magnitude and opposite in direction. b. The charges exert forces on each other equal in magnitude and pointing in the same direction. c. The second charge exerts a larger force on the first charge. d. The first charge exerts a larger force on the second charge. Feedback The correct answer is: The charges exert forces on each other equal in magnitude and opposite in direction. Question 21 Not answered Marked out of 1.00 Not flaggedFlag question Question text Two charges are separated by a distance d and exert mutual attractive forces of F on each other. If the charges are separated by a distance of d/3, what are the new mutual forces? Select one: a. F/9 b. 3F c. F/3 d. 9F Feedback The correct answer is: 9F Question 22 Not answered Marked out of 1.00 Not flaggedFlag question Question text Two charges separated by one meter exert 1-N forces on each other. If the charges are pushed to 1/4 meter separation, the force on each charge will be Select one: a. 16 N. b. 1 N. c. 8 N. d. 4 N. e. 2 N. Feedback The correct answer is: 16 N. Question 23 Not answered Marked out of 1.00 Not flaggedFlag question Question text Two equal and opposite charges a certain distance apart are called an electric "dipole". A positive test charge mc028-1.jpg is placed as shown, equidistant from the two charges. mc028-2.jpg Which diagram below gives the direction of the net force on the test charge? Select one: a. mc028-6.jpg b. mc028-4.jpg c. mc028-3.jpg d. mc028-5.jpg Feedback The correct answer is: mc028-6.jpg Question 24 Not answered Marked out of 1.00 Not flaggedFlag question Question text Two identically charged balls are a certain distance apart. The vectors in the diagram below show the magnitude and direction of the electrostatic force on each ball. mc029-1.jpg Suppose the charge on the left ball is now doubled (represented by two plus signs). Which diagram below best represents the forces that now act on the two balls? Select one: a. mc029-5.jpg b. mc029-2.jpg c. mc029-3.jpg d. mc029-4.jpg Feedback The correct answer is: mc029-2.jpg Question 25 Not answered Marked out of 1.00 Not flaggedFlag question Question text Two protons attract each other gravitationally and repel each other electrically. By far the greater force is Select one: a. answer depends upon the charge of the proton b. neither - the forces are the same. c. the gravitational attraction. d. the electrical repulsion. Feedback The correct answer is: the electrical repulsion. Question 26 Not answered Marked out of 1.00 Not flaggedFlag question Question text What are the units of the Coulomb constant k, which appears in Coulomb's law? Select one: a. N/C b. N2m/C2 c. Nm/C d. Nm2/C2 Feedback The correct answer is: Nm2/C2 Question 27 Not answered Marked out of 1.00 Not flaggedFlag question Question text When two bodies are charged, the total charge before and after charging remains the same because of: Select one: a. quantization of charges b. conservation of charges c. law of induction d. Coulomb's law Feedback When two neutral objects are charged, the total charge remains the same because charges are conserved and are not created or destroyed. The correct answer is: conservation of charges Question 28 Not answered Marked out of 1.00 Not flaggedFlag question Question text Which of the following materials is the best conductor of electricity? Select one: a. Dry air b. Glass c. Rubber d. Wet skin Feedback Wet skin is the best conductor of electricity as compared to dry air, rubber, or glass. The correct answer is: Wet skin Question 29 Not answered Marked out of 1.00 Not flaggedFlag question Question text Which of the following statements is true about electric forces? Select one: a. Electric forces cause objects to only repel each other. b. Electric forces have no effect on each other. c. Electric forces cause objects to repel or attract each other. d. Electric forces cause objects to only attract each other. Feedback Electric forces are repulsive between like charged objects and attractive between objects that possess opposite charges. The correct answer is: Electric forces cause objects to repel or attract each other. Question 30 Not answered Marked out of 1.00 Not flaggedFlag question Question text A conductor that is in electrostatic equilibrium has an electric field inside the conductor that Select one: a. is parallel to the surface of the conductor. b. is greatest near the conductor's surface. c. depends on the radius of the conductor. d. is zero. Feedback The correct answer is: is zero. Question 31 Not answered Marked out of 1.00 Not flaggedFlag question Question text At twice the distance from a point charge, the strength of the electric field Select one: a. is four times its original value. b. is one-half its original value. c. is one-fourth its original value. d. is twice its original value. Feedback The correct answer is: is one-fourth its original value. Question 32 Not answered Marked out of 1.00 Not flaggedFlag question Question text Electric field lines near positive point charges Select one: a. circle clockwise. b. circle counter-clockwise. c. radiate outward. d. radiate inward. Feedback The correct answer is: radiate outward. Question 33 Not answered Marked out of 1.00 Not flaggedFlag question Question text Electric field strength depends on Select one: a. Coulomb constant and mass. b. elementary charge and radius. c. charge and mass. d. charge and distance. Feedback The correct answer is: charge and distance. Question 34 Not answered Marked out of 1.00 Not flaggedFlag question Question text In Fig. 21.11, the electric field lines arising from two charges Q1 and Q2 are shown. Figure 21.11 mc006-1.jpg From this drawing we can see that Select one: a. both Q1 and Q2 have the same sign b. the electric field could be zero at P2 c. None of these is true. d. the electric field could be zero at P1 e. ½Q1½>½ Q2½ Feedback The correct answer is: None of these is true. Question 35 Not answered Marked out of 1.00 Not flaggedFlag question Question text Is it possible to have a zero electric field value between a negative and positive charge along the line joining the two charges? Select one: a. Yes, regardless of the magnitude of the two charges. b. cannot be determined without knowing the separation between the two charges c. No, a zero electric field cannot exist between the two charges. d. Yes, if the two charges are equal in magnitude. Feedback The correct answer is: No, a zero electric field cannot exist between the two charges. Question 36 Not answered Marked out of 1.00 Not flaggedFlag question Question text Is it possible to have a zero electric field value between two positive charges along the line joining the two charges? Select one: a. cannot be determined without knowing the separation between the two charges b. No, a zero electric field cannot exist between the two charges. c. Yes, if the two charges are equal in magnitude. d. Yes, regardless of the magnitude of the two charges. Feedback The correct answer is: Yes, regardless of the magnitude of the two charges. Question 37 Not answered Marked out of 1.00 Not flaggedFlag question Question text The diagram shows electric field lines arising from two small charged particles P and Q. mc007-1.jpg Consider the following two statements: i. The charge on P is smaller than the charge on Q. ii. The electrostatic force on P is smaller than that on Q. Which of the above statements are true? Select one: a. Neither i nor ii. b. Only i. c. Only ii. d. Both i and ii. Feedback The correct answer is: Only i. Question 38 Not answered Marked out of 1.00 Not flaggedFlag question Question text The electric field just outside a charged conductor in electrostatic equilibrium is Select one: a. perpendicular to the conductor's surface. b. the same as it is in the center of the conductor. c. zero. d. at its minimum level. Feedback The correct answer is: perpendicular to the conductor's surface. Question 39 Not answered Marked out of 1.00 Not flaggedFlag question Question text The electric field shown mc005-1.jpg FIGURE 16-2 Select one: a. increases down. b. decreases to the right. c. increases to the right. d. decreases down. e. is uniform. Feedback The correct answer is: increases to the right. Question 40 Not answered Marked out of 1.00 Not flaggedFlag question Question text mc013-1.jpg The figure above shows two particles, each with a charge of +Q, that are located at the opposite corners of a square of side d. What is the direction of the net electric field at point P ? Select one: a. mc013-6.jpg b. mc013-5.jpg c. mc013-4.jpg d. mc013-2.jpg e. mc013-3.jpg Feedback The correct answer is: mc013-4.jpg Question 41 Not answered Marked out of 1.00 Not flaggedFlag question Question text mc014-1.jpg Two positive charges of magnitude q are each a distance d from the origin A of a coordinate system as shown above. At which of the following points is the electric field least in magnitude? Select one: a. C b. B c. E d. A e. D Feedback The correct answer is: A Question 42 Not answered Marked out of 1.00 Not flaggedFlag question Question text Where is the electric field of an isolated, uniformly charged, hollow metallic sphere greatest? Select one: a. at infinity b. at the sphere's inner surface c. at the sphere's outer surface d. at the center of the sphere Feedback The correct answer is: at the sphere's outer surface Question 43 Not answered Marked out of 1.00 Not flaggedFlag question Question text Which is the most correct statement regarding the drawing of electric field lines? Select one: a. Electric field lines always form closed loops. b. Electric field lines can start on a charge of either polarity. c. Electric field lines never cross each other. d. Electric field lines always connect from one charge to another. Feedback The correct answer is: Electric field lines never cross each other. Question 44 Not answered Marked out of 1.00 Not flaggedFlag question Question text A hydrogen atom consists of a proton and an electron. If the orbital radius of the electron increases, the potential energy of the electron Select one: a. remains the same. b. increases. c. decreases. d. depends on the zero point of the potential. Feedback The correct answer is: increases. Question 45 Not answered Marked out of 1.00 Not flaggedFlag question Question text A metallic sphere of radius 5 cm is charged such that the potential of its surface is 100 V (relative to infinity). Which of the following plots correctly shows the potential as a function of distance from the center of the sphere? mc025-1.jpg mc025-2.jpg mc025-3.jpg mc025-4.jpg Select one: a. Plot Z b. Plot Y c. Plot X d. Plot W Feedback The correct answer is: Plot X Question 46 Not answered Marked out of 1.00 Not flaggedFlag question Question text A negative charge is moved from point A to point B along an equipotential surface. Which of the following statements is true for this case? Select one: a. Work is both required and performed in moving the negative charge from point A to point B. b. No work is required to move the negative charge from point A to point B. c. Work is required to move the negative charge from point A to point B. d. The negative charge performs work in moving from point A to point B. Feedback The correct answer is: No work is required to move the negative charge from point A to point B. Question 47 Not answered Marked out of 1.00 Not flaggedFlag question Question text A negative charge, if free, tries to move Select one: a. away from infinity. b. toward infinity. c. from high potential to low potential. d. in the direction of the electric field. e. from low potential to high potential. Feedback The correct answer is: from low potential to high potential. Question 48 Not answered Marked out of 1.00 Not flaggedFlag question Question text A small charged ball is accelerated from rest to a speed v by a 500 V potential difference. If the potential difference is changed to 2000 V, what will the new speed of the ball be? Select one: a. v b. 16v c. 4v d. 2v Feedback The correct answer is: 2v Question 49 Not answered Marked out of 1.00 Not flaggedFlag question Question text A surface on which all points are at the same potential is referred to as Select one: a. an equipotential surface. b. a dielectric surface. c. an equi-voltage surface. d. a constant electric force surface. e. a constant electric field surface. Feedback The correct answer is: an equipotential surface. Question 50 Not answered Marked out of 1.00 Not flaggedFlag question Question text An equipotential surface must be Select one: a. parallel to the electric field at any point. b. randomly oriented with respect to the electric field. c. equal to the electric field at any point. d. perpendicular to the electric field at any point. Feedback The correct answer is: perpendicular to the electric field at any point. Question 51 Not answered Marked out of 1.00 Not flaggedFlag question Question text For a proton moving in the direction of the electric field Select one: a. its potential energy increases and its electric potential increases. b. both its potential energy and it electric potential remain constant. c. its potential energy decreases and its electric potential increases. d. its potential energy decreases and its electric potential decreases. e. its potential energy increases and its electric potential decreases. Feedback The correct answer is: its potential energy decreases and its electric potential decreases. Question 52 Not answered Marked out of 1.00 Not flaggedFlag question Question text For an electron moving in a direction opposite to the electric field Select one: a. its potential energy decreases and its electric potential increases. b. its potential energy increases and its electric potential decreases. c. its potential energy increases and its electric potential increases. d. its potential energy decreases and its electric potential decreases. e. both its potential energy and it electric potential remain constant. Feedback The correct answer is: its potential energy decreases and its electric potential increases. Question 53 Not answered Marked out of 1.00 Not flaggedFlag question Question text One joule per coulomb is a Select one: a. newton. b. farad. c. electron-volt. d. volt. Feedback The correct answer is: volt. Question 54 Not answered Marked out of 1.00 Not flaggedFlag question Question text Several electrons are placed on a hollow conducting sphere. They Select one: a. become uniformly distributed on the sphere's inner surface. b. clump together on the sphere's inner surface. c. become uniformly distributed on the sphere's outer surface. d. clump together on the sphere's outer surface. e. become randomly distributed on the sphere's outer and inner surfaces. Feedback The correct answer is: become uniformly distributed on the sphere's outer surface. Question 55 Not answered Marked out of 1.00 Not flaggedFlag question Question text Suppose a region of space has a uniform electric field, directed towards the right, as shown below. Which statement is true? mc024-1.jpg Select one: a. The voltage at point A is the highest, the voltage at point B is the second highest, and the voltage at point C is the lowest. b. The voltage at all three locations is the same. c. The voltage at points A and B are equal, and the voltage at point C is higher than the voltage at point A. d. None of the above e. The voltage at points A and B are equal, and the voltage at point C is lower than the voltage at point A. Feedback The correct answer is: The voltage at points A and B are equal, and the voltage at point C is lower than the voltage at point A. Question 56 Not answered Marked out of 1.00 Not flaggedFlag question Question text The absolute potential at a distance of 2.0 m from a negative point charge is -100 V. What is the absolute potential 4.0 m away from the same point charge? Select one: a. -400 V b. -25 V c. -200 V d. -50 V Feedback The correct answer is: -50 V Question 57 Not answered Marked out of 1.00 Not flaggedFlag question Question text The absolute potential at a distance of 2.0 m from a positive point charge is 100 V. What is the absolute potential 4.0 m away from the same point charge? Select one: a. 400 V b. 25 V c. 200 V d. 50 V Feedback The correct answer is: 50 V Question 58 Not answered Marked out of 1.00 Not flaggedFlag question Question text The absolute potential at the center of a square is 3.0 V when a charge of +Q is located at one of the square's corners. What is the absolute potential at the square's center when a second charge of -Q is placed at one of the remaining corners? Select one: a. 6.0 V b. 3.0 V c. 9.0 V d. zero Feedback The correct answer is: zero Question 59 Not answered Marked out of 1.00 Not flaggedFlag question Question text The absolute potential at the exact center of a square is 3.0 V when a charge of +Q is located at one of the square's corners. What is the absolute potential at the square's center when each of the other corners is also filled with a charge of +Q? Select one: a. 12 V b. 3.0 V c. 9.0 V d. zero Feedback The correct answer is: 12 V Question 60 Not answered Marked out of 1.00 Not flaggedFlag question Question text The electron-volt is a unit of Select one: a. electric force. b. potential. c. energy. d. charge. e. electric field. Feedback The correct answer is: energy. Question 61 Not answered Marked out of 1.00 Not flaggedFlag question Question text The equipotential surfaces between two spherical conductors are shown in the figure below, with the value of the potential marked for each line. What is the direction of the electric field at point F? mc010-1.jpg Select one: a. towards A b. towards G c. none of the above d. towards E e. towards D Feedback The correct answer is: towards G Question 62 Not answered Marked out of 1.00 Not flaggedFlag question Question text The work done in moving a positive charge against an electric field does not depend on the path chosen in moving the charge in that field. Based on the statement, what kind of force field is the electrostatic field? Select one: a. discrete b. conservative c. nonconservative d. quantized e. polarized Feedback The correct answer is: conservative Question 63 Not answered Marked out of 1.00 Not flaggedFlag question Question text Two electric charges +Q and -Q, are separated by a distance d. If you make a graph of the electric potential as a function of the distance along the line connecting the two charges, the point exactly midway between the two charges would Select one: a. be a relative minimum. b. oscillate between being a relative maximum and a relative minimum. c. be neither a relative maximum nor a relative minimum. d. be a relative maximum. Feedback The correct answer is: be neither a relative maximum nor a relative minimum. Question 64 Not answered Marked out of 1.00 Not flaggedFlag question Question text Two electric charges each equal to -Q, are separated by a distance d. If you make a graph of the electric potential as a function of the distance along the line connecting the two charges, the point exactly midway between the two charges would Select one: a. be a relative maximum. b. be a relative minimum. c. oscillate between being a relative maximum and a relative minimum. d. be neither a relative maximum nor a relative minimum. Feedback The correct answer is: be a relative maximum. Question 65 Not answered Marked out of 1.00 Not flaggedFlag question Question text Two electric charges each equal to +Q, are separated by a distance d. If you make a graph of the electric potential as a function of the distance along the line connecting the two charges, the point exactly midway between the two charges would Select one: a. be a relative minimum. b. be a relative maximum. c. be neither a relative maximum nor a relative minimum. d. oscillate between being a relative maximum and a relative minimum. Feedback The correct answer is: be a relative minimum. Question 66 Not answered Marked out of 1.00 Not flaggedFlag question Question text Two identical aluminum objects are insulated from their surroundings. Object A has a net charge of excess electrons. Object B is grounded. Which object is at a higher potential? Select one: a. A b. Both are at the same potential. c. B d. cannot be determined without more information Feedback The correct answer is: B Question 67 Not answered Marked out of 1.00 Not flaggedFlag question Question text Two parallel metal plates A and B carry positive and negative charges of equal strength, respectively, as shown in the figure below. What is the correct direction for the equipotential lines for this charge configuration? mc006-1.jpg Choice I: vertical, along the y-axis Choice II: horizontal, along the x-axis Select one: a. Choice I b. both Choice I and Choice II c. Choice II d. neither Choice I nor Choice II Feedback The correct answer is: Choice II Question 68 Not answered Marked out of 1.00 Not flaggedFlag question Question text Which of the following is not a vector? Select one: a. electric potential b. electric force c. electric field d. electric line of force Feedback The correct answer is: electric potential

electricity

Atmospheric pressure is approximately Select one: a. 1.01x103 Pa b. 1.01x104 Pa c. 1.01x106 Pa d. 1.01x102 Pa e. 1.01x105 Pa The correct answer is: 1.01x105 Pa Question 2 Density is Select one: a. inversely proportional to mass and proportional to volume. b. proportional to both mass and volume. c. proportional to mass and inversely proportional to volume. d. inversely proportional to both mass and volume. The correct answer is: proportional to mass and inversely proportional to volume. Question 3 One day, while swimming below the surface of the ocean, you let out a number of bubbles of air from your mouth. As the bubbles rise toward the surface, their diameters will Select one: a. increase for half of the bubbles and decrease for the other half. b. increase. c. stay the same. d. decrease. The correct answer is: increase. Question 4 Pressure is Select one: a. inversely proportional to both force and area. b. proportional to both force and area. c. proportional to force and inversely proportional to area. d. inversely proportional to force and proportional to area. The correct answer is: proportional to force and inversely proportional to area. Question 5 Not answered Marked out of 1.00 Not flaggedFlag question Question text Pressure is Select one: a. proportional to both force and area. b. proportional only to area. c. proportional to force and inversely proportional to area. d. inversely proportional to force and proportional to area. e. inversely proportional to both force and area. Feedback The correct answer is: proportional to force and inversely proportional to area. Question 6 Not answered Marked out of 1.00 Not flaggedFlag question Question text The SI unit of pressure is Select one: a. None of these is correct. b. N/m2 c. Pa. d. kg/m/s2 e. All of these are correct. Feedback The correct answer is: All of these are correct. Question 7 Not answered Marked out of 1.00 Not flaggedFlag question Question text What is the difference between the pressures inside and outside a tire called? Select one: a. gauge pressure b. atmospheric pressure c. absolute pressure d. N/m2 Feedback The correct answer is: gauge pressure Question 8 Not answered Marked out of 1.00 Not flaggedFlag question Question text When atmospheric pressure changes, what happens to the absolute pressure at the bottom of a pool? Select one: a. It does not change. b. It increases by a lesser amount. c. It increases by the same amount. d. It increases by a greater amount. Feedback The correct answer is: It increases by the same amount. Question 9 Not answered Marked out of 1.00 Not flaggedFlag question Question text When you hold your hand palm-up, the atmosphere exerts a force of about 800 N on the palm of your hand. Comparing this force with I. the force on the back of the hand. II. the force on the palm when you are holding your hand vertical. III. the force on the palm when it is palm-down. Select one: a. It is equal to II and III but different from I. b. It is equal to I but different from II and III. c. It is equal to II but different from I and III. d. It is different from I, II, and III. e. They are all equal. Feedback The correct answer is: They are all equal. Question 10 Not answered Marked out of 1.00 Not flaggedFlag question Question text Which of the following is not a unit of pressure? Select one: a. N/m b. Pascal c. atmosphere d. mm of mercury Feedback The correct answer is: N/m Question 11 Not answered Marked out of 1.00 Not flaggedFlag question Question text You are originally 1.0 m beneath the surface of a pool. If you dive to 2.0 m beneath the surface, what happens to the absolute pressure on you? Select one: a. It more than doubles. b. It quadruples. c. It less than doubles. d. It doubles. Feedback The correct answer is: It less than doubles. Question 12 Not answered Marked out of 1.00 Not flaggedFlag question Question text "An external pressure applied to an enclosed fluid is transmitted unchanged to every point within the fluid" is known as Select one: a. Pascal's principle. b. Fermet's principle. c. Bernoulli's principle. d. Torricelli's law. e. Archimedes' principle. Feedback The correct answer is: Pascal's principle. Question 13 Not answered Marked out of 1.00 Not flaggedFlag question Question text A 492-N weight sits on the small piston of a hydraulic machine. The small piston has area 3.9 cm2. If the large piston has area 49 cm2, how much weight can the large piston support? Select one: a. 6182 N b. 25 N c. 500 N d. 40000 N Feedback mc008-1.jpg The correct answer is: 6182 N Question 14 Not answered Marked out of 1.00 Not flaggedFlag question Question text A brick weighs 50.0 N, and measures 30.0 cm x 11.7 cm x 4.05 cm. What is the maximum pressure it can exert on a horizontal surface? Select one: a. 11 kPa b. 1.25 Pa c. 12.5 Pa d. 1.25 kPa Feedback mc007-1.jpg The correct answer is: 11 kPa Question 15 Not answered Marked out of 1.00 Not flaggedFlag question Question text A hydraulic press multiplies a force by 100. This multiplication is done at the expense of Select one: a. the mechanism providing the force. b. none of these c. energy, which is divided by 100. d. the distance through which the force acts. e. the time through which the force acts, which is multiplied by 100. Feedback The correct answer is: the distance through which the force acts. Question 16 Not answered Marked out of 1.00 Not flaggedFlag question Question text A plastic block of dimensions 2.00 cm x 3.00 cm x 4.00 cm has a mass of 37.8g. What is its density? Select one: a. 1.60 g/cm3 b. 0.80 g/cm3 c. 1.58 g/cm3 d. 1.20 g/cm3 Feedback The correct answer is: 1.58 g/cm3 Question 17 Not answered Marked out of 1.00 Not flaggedFlag question Question text At a depth d in the ocean, the gauge pressure is p. The depth at which the gauge pressure would be three times as great is: Select one: a. 9d b. 6d c. mc021-1.jpg d d. mc021-2.jpg d e. 3d Feedback The correct answer is: 3d Question 18 Not answered Marked out of 1.00 Not flaggedFlag question Question text Consider three drinking glasses. All three have the same area base, and all three are filled to the same depth with water. Glass A is cylindrical. Glass B is wider at the top than at the bottom, and so holds more water than A. Glass C is narrower at the top than at the bottom, and so holds less water than A. Which glass has the greatest liquid pressure at the bottom? Select one: a. Glass C b. Glass A c. Glass B d. All three have equal pressure. Feedback The correct answer is: All three have equal pressure. Question 19 Not answered Marked out of 1.00 Not flaggedFlag question Question text Each of the three containers shown in Figure 15-1 weighs the same amount and is filled with water to the same level. Figure 15-1 mc017-1.jpg All three have the same surface area in contact with the table. Which of the following statements are true? I. The pressure at the bottom surface of the container is the same for all three containers. II. The force exerted by the water on the bottom surface of the container is the same for all three containers. III. The force exerted by the container on the table is the same for all three containers. Select one: a. Only Statements II and III are true. b. Only Statements I and III are true. c. None of the statements are true. d. Statements I, II, and III are true. e. Only Statements I and II are true. Feedback The correct answer is: Only Statements I and II are true. Question 20 Not answered Marked out of 1.00 Not flaggedFlag question Question text For incompressible fluids, density changes little with changes in Select one: a. depth. b. free-fall acceleration. c. temperature. d. pressure. Feedback The correct answer is: pressure. Question 21 Not answered Marked out of 1.00 Not flaggedFlag question Question text How does a liquid differ from a gas? Select one: a. A liquid has definite shape, unlike a gas. b. A liquid has definite shape, whereas a gas has definite volume. c. A liquid has definite volume, unlike a gas. d. A liquid has both definite shape and definite volume, whereas a gas has neither. Feedback The correct answer is: A liquid has definite volume, unlike a gas. Question 22 Not answered Marked out of 1.00 Not flaggedFlag question Question text In a hydraulic garage lift, the small piston has a radius of 4.3 cm and the large piston has a radius of 15 cm. What force must be applied on the small piston in order to lift a car weighing 19732 N on the large piston? Select one: a. 2.9x103 N b. 6.7x103 N c. 5.0x103 N d. 1.6x103 N Feedback mc009-1.jpg The correct answer is: 1.6x103 N Question 23 Not answered Marked out of 1.00 Not flaggedFlag question Question text In Fig. 14.5, fluid fills the container shown here. At which of the indicated points is the pressure greatest? Figure 14.5 mc020-1.jpg Select one: a. A b. B c. D d. The pressure is the same at each of the labeled points. e. C Feedback The correct answer is: The pressure is the same at each of the labeled points. Question 24 Not answered Marked out of 1.00 Not flaggedFlag question Question text Salt water is more dense than fresh water. A ship floats in both fresh water and salt water. Compared to the fresh water, the volume of water displaced in the salt water is Select one: a. the same. b. cannot be determined from the information given c. more. d. less. Feedback The correct answer is: less. Question 25 Not answered Marked out of 1.00 Not flaggedFlag question Question text Substance A has a density of 3.0 g/cm3 and substance B has a density of 4.0 g/cm3. In order to obtain equal masses of these two substances, the ratio of the volume of A to the volume of B will be equal to Select one: a. 4:3. b. 1:3. c. 1:4. d. 3:4. Feedback The correct answer is: 4:3. Question 26 Not answered Marked out of 1.00 Not flaggedFlag question Question text Substance A has a density of 3.0 g/cm3 and substance B has a density of 4.0 g/cm3. In order to obtain equal masses of these two substances, the ratio of the volume of A to the volume of B will be equal to Select one: a. 3:4. b. 1:4. c. 4:3. d. 1:3. Feedback The correct answer is: 4:3. Question 27 Not answered Marked out of 1.00 Not flaggedFlag question Question text The Aswan High Dam is 111 m high. What is the gauge water pressure at the foot of the dam? Select one: a. 1.11x102 Pa b. 1.09x106 Pa c. 1.09x103 Pa d. 1.11x105 Pa e. 1.16x106 Pa Feedback The correct answer is: 1.09x106 Pa Question 28 Not answered Marked out of 1.00 Not flaggedFlag question Question text The hydraulic lift is an application of Select one: a. Pascal's principle. b. Poiseuille's equation. c. Bernoulli's principle. d. Torricelli's equation. e. Archimedes' principle. Feedback The correct answer is: Pascal's principle. Question 29 Not answered Marked out of 1.00 Not flaggedFlag question Question text The pressure at the bottom of a jug filled with water does NOT depend on Select one: a. water density. b. surface area of the water. c. none of these d. the acceleration due to gravity. e. the height of the liquid. Feedback The correct answer is: surface area of the water. Question 30 Not answered Marked out of 1.00 Not flaggedFlag question Question text The pressure in a liquid depends on liquid Select one: a. both of these b. depth. c. density. d. neither of these Feedback The correct answer is: both of these Question 31 Not answered Marked out of 1.00 Not flaggedFlag question Question text The ratio of output force to input force of a hydraulic press will be equal to the ratio of the output and input piston Select one: a. radii. b. areas. c. diameters. d. none of these e. all of these Feedback The correct answer is: areas. Question 32 Not answered Marked out of 1.00 Not flaggedFlag question Question text The tires of a car support the weight of a stationary car. If one tire has a slow leak, the air pressure within the tire will _______ with time, the surface area between the tire and the road will ________ in time, and the net force the tire exerts on the road will ___________ in time. Select one: a. decrease, increase, decrease b. increase, increase, increase c. decrease, decrease, decrease d. decrease, increase, increase e. decrease, increase, remain constant Feedback The correct answer is: decrease, increase, remain constant Question 33 Not answered Marked out of 1.00 Not flaggedFlag question Question text The U-tube shown in Figure 15-3 contains mercury with a density of 13,600 kg/m3. It is open on one end and the other end is connected to a boiler, which contains water at high pressure. The water from the boiler pushes directly on the mercury column. Figure 15-3 mc018-1.jpg What is the gauge pressure at point A in the boiler? Select one: a. 441 Pa b. 4.41 kPa c. 449 Pa d. 128 Pa e. 4.54 kPa Feedback The correct answer is: 4.41 kPa Question 34 Not answered Marked out of 1.00 Not flaggedFlag question Question text To multiply the input force of a hydraulic lift , the input end should be the one having the Select one: a. larger diameter piston. b. Relative piston sizes don't matter. c. smaller diameter piston. Feedback The correct answer is: smaller diameter piston. Question 35 Not answered Marked out of 1.00 Not flaggedFlag question Question text What does the net force between two levels in a fluid equal? Select one: a. the force applied to the fluid's surface b. the weight of the fluid above the top level c. the weight of the fluid between the levels d. the force applied to the fluid's sides Feedback The correct answer is: the weight of the fluid between the levels Question 36 Not answered Marked out of 1.00 Not flaggedFlag question Question text When holes are drilled through the wall of a water tower , water will spurt out the greatest horizontal distance from the hole closest to Select one: a. the middle of the tower. b. the bottom of the tower. c. The horizontal distance will be the same for all holes. d. the top of the tower. Feedback The correct answer is: the bottom of the tower. Question 37 Not answered Marked out of 1.00 Not flaggedFlag question Question text Which of the following is a fluid? Select one: a. helium b. gold c. ice d. iron Feedback The correct answer is: helium Question 38 Not answered Marked out of 1.00 Not flaggedFlag question Question text Which of the following is not a fluid? Select one: a. seawater b. wood c. carbon dioxide d. hydrogen Feedback The correct answer is: wood Question 39 Not answered Marked out of 1.00 Not flaggedFlag question Question text 50 cm3 of wood is floating on water, and 50 cm3 of iron is totally submerged. Which has the greater buoyant force on it? Select one: a. cannot be determined without knowing their densities b. the wood c. the iron d. Both have the same buoyant force. Feedback The correct answer is: the iron Question 40 Not answered Marked out of 1.00 Not flaggedFlag question Question text A 10-kg piece of aluminum sits at the bottom of a lake, right next to a 10-kg piece of lead. Which has the greater buoyant force on it? Select one: a. cannot be determined without knowing their volumes b. Both have the same buoyant force. c. the aluminum d. the lead Feedback The correct answer is: the aluminum Question 41 Not answered Marked out of 1.00 Not flaggedFlag question Question text A 4.57-kg cylinder of solid iron is supported by a string while submerged in water. What is the tension in the string? (The density of iron is 7860kg/m3.) Select one: a. 19.6 N b. 2.50 N c. 39.1 N d. 23.7 N Feedback mc014-1.jpg The correct answer is: 39.1 N Question 42 Not answered Marked out of 1.00 Not flaggedFlag question Question text A block of metal weighs 40 N in air and 30 N in water. What is the buoyant force of the water? Select one: a. 10 N b. 30 N c. 70 N d. 40 N Feedback The correct answer is: 10 N Question 43 Not answered Marked out of 1.00 Not flaggedFlag question Question text A boat loaded with rocks is floating in a swimming pool. If the rocks are thrown into the pool, the water level in the pool, after the rocks have settled to the bottom, Select one: a. stays the same. b. falls. c. rises. d. There is not enough information to answer this question. Feedback The correct answer is: falls. Question 44 Not answered Marked out of 1.00 Not flaggedFlag question Question text A cup of water is filled to the brim when an ice cube is placed in it. The tip of the ice cube sticks out of the surface. As the ice melts, you observe that Select one: a. the cup might overflow but it depends on the actual mass of the ice cube. b. There is not enough information to answer this question. c. the cup overflows. d. the water level remains the same. e. the water level actually goes down. Feedback The correct answer is: the water level remains the same. Question 45 Not answered Marked out of 1.00 Not flaggedFlag question Question text A flask of water rests on a scale. If you dip your finger into the water, without touching the flask, the reading on the scale will Select one: a. decrease. b. increase. c. stay the same. d. behave unpredictably. Feedback The correct answer is: increase. Question 46 Not answered Marked out of 1.00 Not flaggedFlag question Question text A piece of iron rests on top of a piece of wood floating in a bathtub. If the iron is removed from the wood, what happens to the water level in the tub? Select one: a. impossible to determine from the information given b. It goes down. c. It goes up. d. It does not change. Feedback The correct answer is: It goes down. Question 47 Not answered Marked out of 1.00 Not flaggedFlag question Question text A piece of wood is floating in a bathtub. A second piece of wood sits on top of the first piece, and does not touch the water. If the top piece is taken off and placed in the water, what happens to the water level in the tub? Select one: a. It does not change. b. cannot be determined from the information given c. It goes up. d. It goes down. Feedback The correct answer is: It does not change. Question 48 Not answered Marked out of 1.00 Not flaggedFlag question Question text A rock is suspended from a scale reads 10.0 N. A beaker of water is raised up so the rock is totally submerged in the water. The scale now reads 6.25 N. What is the density of the rock? Select one: a. 2.50 times the density of water b. 2.00 times the density of water c. 2.33 times the density of water d. 3.00 times the density of water e. 2.67 times the density of water Feedback The correct answer is: 2.67 times the density of water Question 49 Not answered Marked out of 1.00 Not flaggedFlag question Question text A solid object floats in water with three-fourths of its volume beneath the surface. What is the object's density? Select one: a. 1333 kg/m3 b. 1000 kg/m3 c. 750 kg/m3 d. 250 kg/m3 Feedback The correct answer is: 750 kg/m3 Question 50 Not answered Marked out of 1.00 Not flaggedFlag question Question text A steel ball sinks in water but floats in a pool of mercury. Where is the buoyant force on the ball greater? Select one: a. submerged in the water b. floating on the mercury c. cannot be determined from the information given d. It is the same in both cases. Feedback The correct answer is: floating on the mercury Question 51 Not answered Marked out of 1.00 Not flaggedFlag question Question text An object has a volume of 4.1 m3 and weighs 69525 N. What will its weight be in water? Select one: a. 29345 N b. 40,000 N c. 39,200 N d. 9,800 N Feedback mc013-1.jpg The correct answer is: 29345 N Question 52 Not answered Marked out of 1.00 Not flaggedFlag question Question text As a rock sinks deeper and deeper into water of constant density, what happens to the buoyant force on it? Select one: a. It may increase or decrease, depending on the shape of the rock. b. It increases. c. It remains constant. d. It decreases. Feedback The correct answer is: It remains constant. Question 53 Not answered Marked out of 1.00 Not flaggedFlag question Question text If the density of gold is 19.3x103 kg/m3, what buoyant force does a 0.53-kg gold crown experience when it is immersed in water? Select one: a. 3.0x10-2 N b. 3.0x10-4 N c. 3.0x10-5 N d. 0.27 N Feedback mc015-1.jpg The correct answer is: 0.27 N Question 54 Not answered Marked out of 1.00 Not flaggedFlag question Question text Salt water has greater density than fresh water. A boat floats in both fresh water and in salt water. Where is the buoyant force greater on the boat? Select one: a. impossible to determine from the information given b. salt water c. Buoyant force is the same in both. d. fresh water Feedback The correct answer is: Buoyant force is the same in both. Question 55 Not answered Marked out of 1.00 Not flaggedFlag question Question text A cylindrical water tower, open to the atmosphere, with a radius of 5.0m, is positioned such that it is 75m above the tap in your kitchen which has a radius of 1.0cm. If you leave you tap open, at what speed does the water level fall in the water tower. Give your answer in mm/s. mc014-1.jpg Select one: a. 38 mm/s b. 0.15 mm/s c. 15 mm/s d. 0.38 mm/s Feedback mc014-2.jpg The correct answer is: 0.15 mm/s Question 56 Not answered Marked out of 1.00 Not flaggedFlag question Question text An ideal fluid flows at 12 m/s in a horizontal pipe. If the pipe narrows to half its original radius, what is the flow speed in the narrower section? Select one: a. 48 m/s b. 12 m/s c. 36 m/s d. 24 m/s Feedback The correct answer is: 48 m/s Question 57 Not answered Marked out of 1.00 Not flaggedFlag question Question text An ideal fluid flows at 12 m/s in a horizontal pipe. If the pipe widens to twice its original radius, what is the flow speed in the wider section? Select one: a. 4.0 m/s b. 3.0 m/s c. 12 m/s d. 6.0 m/s Feedback The correct answer is: 3.0 m/s Question 58 Not answered Marked out of 1.00 Not flaggedFlag question Question text An ideal fluid flows through a pipe made of two sections with diameters of 1 cm and 3 cm, respectively. By what factor would you have to multiply the velocity of the liquid flowing through the 1 cm section to obtain the velocity of liquid flowing through the 3 cm section? Select one: a. 9 b. 6 c. mc015-2.jpg d. mc015-1.jpg Feedback The correct answer is: mc015-2.jpg Question 59 Not answered Marked out of 1.00 Not flaggedFlag question Question text As the speed of a moving fluid increases, the pressure in the fluid Select one: a. increases. b. may increase or decrease, depending on the viscosity. c. decreases. d. remains constant. Feedback The correct answer is: decreases. Question 60 Not answered Marked out of 1.00 Not flaggedFlag question Question text For an ideal fluid flowing through a horizontal pipe, Bernoulli's principle and the continuity equation state that the pressure within the pipe does which of the following? (Assume measurements are taken along the pipe in the direction of fluid flow.) Select one: a. Pressure decreases as the pipe diameter increases. b. Pressure remains constant as the pipe diameter increases. c. Pressure increases, then decreases as the pipe diameter increases. d. Pressure increases as the pipe diameter increases. Feedback The correct answer is: Pressure increases as the pipe diameter increases. Question 61 Not answered Marked out of 1.00 Not flaggedFlag question Question text The Bernoulli effect is responsible for the lift force on an airplane wing. Wings must therefore be designed so as to insure that Select one: a. wings are thick enough to create a significant pressure difference between the top and bottom surfaces of the wings because of the different heights of these surfaces. b. air molecules will be deflected downward when they hit the wing. c. air molecules move more rapidly past the upper surface of the wing than past the lower surface. d. air molecules move more rapidly past the lower surface of the wing than past the upper surface. e. air molecules will be deflected upward when they hit the wing. Feedback The correct answer is: air molecules move more rapidly past the upper surface of the wing than past the lower surface. Question 62 Not answered Marked out of 1.00 Not flaggedFlag question Question text The equation of continuity is a statement of the Select one: a. conservation of linear momentum. b. conservation of angular momentum. c. conservation of mass. d. conservation of energy. e. conservation of kinetic energy. Feedback The correct answer is: conservation of mass. Question 63 Not answered Marked out of 1.00 Not flaggedFlag question Question text The lift on an airplane wing is an application of Select one: a. Torricelli's equation. b. Bernoulli's principle. c. Poiseuille's equation. d. Pascal's principle. e. Archimedes' principle. Feedback The correct answer is: Bernoulli's principle. Question 64 Not answered Marked out of 1.00 Not flaggedFlag question Question text Water flows through a pipe. The diameter of the pipe at point B is larger than at point A. Where is the speed of the water greater? Select one: a. same at both A and B b. point B c. cannot be determined from the information given d. point A Feedback The correct answer is: point A Question 65 Not answered Marked out of 1.00 Not flaggedFlag question Question text Water flows through a pipe. The diameter of the pipe at point B is larger than at point A. Where is the water pressure greatest? Select one: a. cannot be determined from the information given b. point A c. point B d. same at both A and B Feedback The correct answer is: point B Question 66 Not answered Marked out of 1.00 Not flaggedFlag question Question text Water is flowing in a horizontal pipe of diameter d. If you want to change the diameter of this pipe so that the speed of the water would be half as great as it was, the new diameter should be: Select one: a. 2d b. d/4 c. d/2 d. mc011-2.jpg e. mc011-1.jpg Feedback The correct answer is: mc011-2.jpg Question 67 Not answered Marked out of 1.00 Not flaggedFlag question Question text Water is flowing in a pipe. If you double the diameter of the pipe and at the same time increase the pressure so that you double the speed of the water, the volume flow rate will increase by a factor of: Select one: a. 2 b. mc012-1.jpg c. 8 d. 4 e. 16 Feedback The correct answer is: 8 Question 68 Not answered Marked out of 1.00 Not flaggedFlag question Question text When you blow some air above a paper strip, the paper rises. This is because Select one: a. the air above the paper moves slower and the pressure is higher. b. the air above the paper moves faster and the pressure is lower. c. the air above the paper moves faster and the pressure is higher. d. the air above the paper moves slower and the pressure is lower. Feedback The correct answer is: the air above the paper moves faster and the pressure is lower. Question 69 Not answered Marked out of 1.00 Not flaggedFlag question Question text Which one of the following is associated with the law of conservation of energy in fluids? Select one: a. Archimedes' principle b. equation of continuity c. Pascal's principle d. Bernoulli's principle Feedback The correct answer is: Bernoulli's principle Question 70 Not answered Marked out of 1.00 Not flaggedFlag question Question text Why does an ideal fluid move faster through a pipe with decreasing diameter? Select one: a. The pipe exerts more pressure on the fluid. b. The pressure within the fluid increases. c. The fluid moves downhill. d. The pressure within the fluid decreases. Feedback The correct answer is: The pressure within the fluid decreases. Question 71 Not answered Marked out of 1.00 Not flaggedFlag question Question text Why does the lift on an airplane wing increase as the speed of the airplane increases? Select one: a. The pressure above the wing becomes greater than the pressure below the wing. b. The pressure behind the wing becomes less than the pressure in front of the wing. c. The pressure behind the wing becomes greater than the pressure in front of the wing. d. The pressure above the wing becomes less than the pressure below the wing. Feedback The correct answer is: The pressure above the wing becomes less than the pressure below the wing. Question 72 Not answered Marked out of 1.00 Not flaggedFlag question Question text You are driving a late model convertible at the 55 mph limit with its soft ragtop roof up and the windows closed. You observe that the roof Select one: a. is not changed from when the car was at rest. b. bows inward only when you are driving uphill. c. bows inward. d. bows outward. e. bows inward only when you are driving downhill. Feedback Figure 14.3 graphic(14.3.bmp) The correct answer is: bows outward.

fluids

50 g of ice at 0°C is dropped in a beaker containing 100 g of water at 0°C. What will be the contents of the beaker after 5 hours? Assume that the room temperature is 0°C. Select one: a. 25 g of ice and 125 g of water b. 75 g of ice and 75 g of water c. 50 g of ice and 100 g of water d. 150 g of water Feedback The correct answer is: 50 g of ice and 100 g of water Question 2 Not answered Marked out of 1.00 Not flaggedFlag question Question text 5000 J of heat is supplied in one minute to 0.06 kg of a liquid at its boiling point of 85°C to convert it completely to vapor. Which of the following expressions gives the heat of vaporization of the liquid? Select one: a. mc016-3.jpg b. mc016-2.jpg c. mc016-1.jpg d. mc016-4.jpg Feedback The correct answer is: mc016-1.jpg Question 3 Not answered Marked out of 1.00 Not flaggedFlag question Question text A good heat conductor is Select one: a. neither a poor nor a good insulator. b. both a good conductor and a good insulator c. a poor insulator. d. a good insulator. Feedback The correct answer is: a poor insulator. Question 4 Not answered Marked out of 1.00 Not flaggedFlag question Question text A solid is heated at a constant rate until it reaches the vapor state. The temperature of the substance changes with time as shown in the graph below. Which part(s) of the graph indicate(s) that the substance exists in solid-liquid and liquid-vapor state? mc020-1.jpg Select one: a. OP, QR, ST b. OP c. PQ, RS d. OP, ST Feedback The correct answer is: PQ, RS Question 5 Not answered Marked out of 1.00 Not flaggedFlag question Question text nar001-1.jpg At what point on the figure above does the substance (10.0g of H2O) undergo a phase change? Select one: a. A b. B c. E d. C Feedback The correct answer is: B Question 6 Not answered Marked out of 1.00 Not flaggedFlag question Question text nar001-1.jpg During which process in the figure above is the amount of energy transferred as heat to the substance (10.0g of H2O) approximately 4.19 ´ 103 J? Select one: a. D b. C c. A d. B Feedback The correct answer is: C Question 7 Not answered Marked out of 1.00 Not flaggedFlag question Question text Energy transferred as heat occurs between two bodies in thermal contact when they differ in which of the following properties? Select one: a. density b. temperature c. specific heat d. mass Feedback The correct answer is: temperature Question 8 Not answered Marked out of 1.00 Not flaggedFlag question Question text Heat is added to an open container of a liquid. The liquid is brought to its boiling point and half the liquid boils away. Which of the following graphs shows how the temperature changes with time in this period? Select one: a. mc018-3.jpg b. mc018-2.jpg c. mc018-1.jpg d. mc018-4.jpg Feedback The correct answer is: mc018-2.jpg Question 9 Not answered Marked out of 1.00 Not flaggedFlag question Question text How is energy transferred as heat always directed? Select one: a. from an object at high temperature to an object at low temperature b. from an object at low temperature to an object at high temperature c. from an object at low kinetic energy to an object at high kinetic energy d. from an object with higher mass to an object of lower mass Feedback The correct answer is: from an object at high temperature to an object at low temperature Question 10 Not answered Marked out of 1.00 Not flaggedFlag question Question text If two small beakers of water, one at 70°C and one at 80°C, are emptied into a large beaker, what is the final temperature of the water? Select one: a. The water temperature will fluctuate. b. The final temperature is greater than 80°C. c. The final temperature is less than 70°C. d. The final temperature is between 70°C and 80°C. Feedback The correct answer is: The final temperature is between 70°C and 80°C. Question 11 Not answered Marked out of 1.00 Not flaggedFlag question Question text In the formula mc021-1.jpg, which quantity is measured in units of J/g·°C? Select one: a. DT b. c c. m d. Q Feedback The correct answer is: c Question 12 Not answered Marked out of 1.00 Not flaggedFlag question Question text Metals are both good heat conductors and good electrical conductors because of the Select one: a. high elasticity of metals. b. similarity between thermal and electrical conductive properties. c. relatively high densities of metals. d. looseness of outer electrons in metal atoms. e. ability of metals to transfer energy easily. Feedback The correct answer is: looseness of outer electrons in metal atoms. Question 13 Not answered Marked out of 1.00 Not flaggedFlag question Question text Objects that radiate relatively well, Select one: a. neither of these b. reflect radiation relatively well. c. both of these d. absorb radiation relatively well. Feedback The correct answer is: absorb radiation relatively well. Question 14 Not answered Marked out of 1.00 Not flaggedFlag question Question text Place a kilogram block of iron at 40 degrees C into a kilogram of water at 20 degrees C and the final temperature of the two becomes Select one: a. at or about 30 degrees C. b. less than 30 degrees C. c. more than 30 degrees C. d. impossible to calculate Feedback The correct answer is: less than 30 degrees C. Question 15 Not answered Marked out of 1.00 Not flaggedFlag question Question text Pour a liter of water at 40 degrees C into a liter of water at 20 degrees C and the final temperature of the two becomes Select one: a. at or about 30 degrees C. b. impossible to calculate c. more than 30 degrees C. d. less than 30 degrees C. Feedback The correct answer is: at or about 30 degrees C. Question 16 Not answered Marked out of 1.00 Not flaggedFlag question Question text Substances absorb heat energy by the process of Select one: a. convection. b. radiation. c. all of these d. conduction. Feedback The correct answer is: all of these Question 17 Not answered Marked out of 1.00 Not flaggedFlag question Question text nar001-1.jpg The figure above shows how the temperature of 10.0 g of ice changes as energy is added. Which of the following statements is correct? Select one: a. The water absorbed energy continuously, but the temperature increased only when all of the water was in one phase. b. The water absorbed energy continuously, and the temperature increased continuously. c. The water absorbed energy sporadically, and the temperature increased only when all of the water was in one phase. d. The water did not absorb energy. Feedback The correct answer is: The water absorbed energy continuously, but the temperature increased only when all of the water was in one phase. Question 18 Not answered Marked out of 1.00 Not flaggedFlag question Question text The planet Earth loses heat mainly by Select one: a. radiation. b. conduction. c. convection. d. all of these Feedback Figure 9-B mc029-1.jpg The correct answer is: radiation. Question 19 Not answered Marked out of 1.00 Not flaggedFlag question Question text The use of fiberglass insulation in the outer walls of a building is intended to minimize heat transfer through what process? Select one: a. convection b. conduction c. radiation d. vaporization Feedback The correct answer is: conduction Question 20 Not answered Marked out of 1.00 Not flaggedFlag question Question text To melt 4 kg of a solid, 106 J of heat is required. Which of the following expressions gives the heat of fusion of the solid? Select one: a. mc015-3.jpg b. mc015-2.jpg c. mc015-4.jpg d. mc015-1.jpg Feedback The correct answer is: mc015-3.jpg Question 21 Not answered Marked out of 1.00 Not flaggedFlag question Question text To which of the following is high temperature related? Select one: a. zero net energy transfer b. high particle kinetic energy c. large volume d. low particle kinetic energy Feedback The correct answer is: high particle kinetic energy Question 22 Not answered Marked out of 1.00 Not flaggedFlag question Question text nar001-1.jpg Using the figure above, determine which value equals the latent heat required to change the liquid (10.0g of H2O) water into steam. Select one: a. 30.6 x 106 J b. 31.1 x 106 J c. 8.04 x 106 J d. 22.6 x 106 J Feedback The correct answer is: 22.6 x 106 J Question 23 Not answered Marked out of 1.00 Not flaggedFlag question Question text What happens when water at its melting point solidifies to ice without any change in temperature? Select one: a. Latent heat is emitted. b. Latent heat is absorbed. c. Specific heat is emitted. d. Both latent heat and specific heat are emitted. Feedback The correct answer is: Latent heat is emitted. Question 24 Not answered Marked out of 1.00 Not flaggedFlag question Question text When a gas is changed to a liquid state, the gas Select one: a. neither releases nor absorbs energy. b. both releases and absorbs energy. c. absorbs energy. d. releases energy. Feedback The correct answer is: releases energy. Question 25 Not answered Marked out of 1.00 Not flaggedFlag question Question text When a solid is changed to a liquid state, the solid Select one: a. absorbs energy. b. both releases and absorbs energy. c. neither releases nor absorbs energy. d. releases energy. Feedback The correct answer is: absorbs energy. Question 26 Not answered Marked out of 1.00 Not flaggedFlag question Question text Which of the following best describes the relationship between two systems in thermal equilibrium? Select one: a. No net energy is exchanged. b. The masses are equal. c. The velocity is zero. d. The volumes are equal. Feedback The correct answer is: No net energy is exchanged. Question 27 Not answered Marked out of 1.00 Not flaggedFlag question Question text Which of the following is a direct cause of a substance's temperature increase? Select one: a. Energy is removed from the particles of the substance. b. The volume of the substance decreases. c. Kinetic energy is added to the particles of the substance. d. The number of atoms and molecules in a substance changes. Feedback The correct answer is: Kinetic energy is added to the particles of the substance. Question 28 Not answered Marked out of 1.00 Not flaggedFlag question Question text Which of the following is proportional to the kinetic energy of atoms and molecules? Select one: a. potential energy b. elastic energy c. temperature d. thermal equilibrium Feedback The correct answer is: temperature Question 29 Not answered Marked out of 1.00 Not flaggedFlag question Question text Which of the following is true during a phase change? Select one: a. Temperature decreases. b. Temperature increases. c. Temperature remains constant. d. There is no transfer of energy as heat. Feedback The correct answer is: Temperature remains constant. Question 30 Not answered Marked out of 1.00 Not flaggedFlag question Question text Which of the following physical properties can be used to measure temperature using a thermometer? Select one: a. pressure b. volume c. viscosity d. density Feedback The correct answer is: volume Question 31 Not answered Marked out of 1.00 Not flaggedFlag question Question text Your feet feel warmer on a rug than on your tile floor because your rug Select one: a. none of these b. all of these c. is usually warmer than your tile. d. for the same mass has more internal energy than your tile. e. is a better insulator than your tile. Feedback The correct answer is: is a better insulator than your tile. Question 32 Not answered Marked out of 1.00 Not flaggedFlag question Question text A balloon is filled with 500 cubic centimeters of air at 27ºC. To what temperature must the air be changed to reduce the volume of the balloon to 250 cubic centimeters? Select one: a. 13.5ºC b. 54ºC c. 150ºC d. -123ºC Feedback The correct answer is: -123ºC Question 33 Not answered Marked out of 1.00 Not flaggedFlag question Question text A container holds N molecules of an ideal gas at a given temperature. If the number of molecules in the container is increased to 2N with no change in temperature or volume, the pressure in the container Select one: a. none of the above b. doubles. c. is cut in half. d. remains constant. Feedback The correct answer is: doubles. Question 34 Not answered Marked out of 1.00 Not flaggedFlag question Question text A container of an ideal gas at 1 atm is compressed to one-third its volume, with the temperature held constant. What is its final pressure? Select one: a. 1/3 atm b. 9 atm c. 1 atm d. 3 atm Feedback The correct answer is: 3 atm Question 35 Not answered Marked out of 1.00 Not flaggedFlag question Question text A fixed container holds oxygen and helium gases at the same temperature. Which one of the following statements is correct? Select one: a. The helium molecules have the greater kinetic energy. b. The helium molecules have the greater speed. c. The oxygen molecules have the greater kinetic energy. d. The oxygen molecules have the greater speed. Feedback The correct answer is: The helium molecules have the greater speed. Question 36 Not answered Marked out of 1.00 Not flaggedFlag question Question text A mole of diatomic oxygen molecules and a mole of diatomic nitrogen molecules at STP have Select one: a. all of the above b. the same number of molecules. c. the same average molecular speeds. d. the same diffusion rates. Feedback The correct answer is: the same number of molecules. Question 37 Not answered Marked out of 1.00 Not flaggedFlag question Question text A sample of an ideal gas is heated and its Kelvin temperature doubles. What happens to the average speed of the molecules in the sample? Select one: a. It doubles. b. It halves. c. It is mc027-1.jpg times as large. d. It is 1/mc027-2.jpg times smaller. e. It does not change. Feedback The correct answer is: It is mc027-1.jpg times as large. Question 38 Not answered Marked out of 1.00 Not flaggedFlag question Question text A sample of an ideal gas is in a tank of constant volume. The sample absorbs heat energy so that its temperature changes from 300 K to 600 K. If v1 is the average speed of the gas molecules before the absorption of heat and v2 is their average speed after the absorption of heat, what is the ratio v2/ v1 ? Select one: a. mc020-1.jpg b. 4 c. 1 d. 1/2 e. 2 Feedback The correct answer is: mc020-1.jpg Question 39 Not answered Marked out of 1.00 Not flaggedFlag question Question text A sample of an ideal gas is slowly compressed to one-half its original volume with no change in temperature. What happens to the average speed of the molecules in the sample? Select one: a. It doubles. b. none of the above c. It halves. d. It does not change. Feedback The correct answer is: It does not change. Question 40 Not answered Marked out of 1.00 Not flaggedFlag question Question text A sample of an ideal gas is slowly compressed to one-half its original volume with no change in temperature. What happens to the average speed of the molecules in the sample? Select one: a. It halves. b. It quadruples. c. cannot be determined without more information d. It does not change. e. It doubles. Feedback The correct answer is: It does not change. Question 41 Not answered Marked out of 1.00 Not flaggedFlag question Question text According to the ideal gas Law, PV = constant for a given temperature. As a result, an increase in volume corresponds to a decrease in pressure. This happens because the molecules Select one: a. collide with each other more frequently. b. move slower on the average. c. strike the container wall less often. d. transfer less energy to the walls of the container each time they strike it. Feedback The correct answer is: strike the container wall less often. Question 42 Not answered Marked out of 1.00 Not flaggedFlag question Question text Both the pressure and volume of a given sample of an ideal gas double. This means that its temperature in Kelvin must Select one: a. quadruple. b. remain unchanged. c. double. d. reduce to one-fourth its original value. Feedback The correct answer is: quadruple. Question 43 Not answered Marked out of 1.00 Not flaggedFlag question Question text How many atoms are there in a mole of Helium (He) gas? Select one: a. 1.20´1024 b. 3.01´1023 c. 6.02´1023 d. cannot be determined with out the molar mass Feedback The correct answer is: 6.02´1023 Question 44 Not answered Marked out of 1.00 Not flaggedFlag question Question text How many atoms are there in a mole of Hydrogen (H2) gas? Select one: a. 6.02´1023 b. cannot be determined with out the molar mass c. 1.20´1024 d. 3.01´1023 Feedback The correct answer is: 1.20´1024 Question 45 Not answered Marked out of 1.00 Not flaggedFlag question Question text If the pressure acting on an ideal gas at constant temperature is tripled, its volume is Select one: a. increased by a factor of two. b. reduced to one-half. c. increased by a factor of three. d. reduced to one-third. Feedback The correct answer is: reduced to one-third. Question 46 Not answered Marked out of 1.00 Not flaggedFlag question Question text In nature, heat will always flow from a: Select one: a. cold object to the warm object. b. small object to the large object. c. large object to the small object. d. warm object to the cold object. Feedback The correct answer is: warm object to the cold object. Question 47 Not answered Marked out of 1.00 Not flaggedFlag question Question text On a cold winter day, you turn up the thermostat in your house. Assume that your house is well sealed and that no air enters or leaves the house. When the air temperature in your house rises a short while later, what statement is correct regarding the air pressure? Select one: a. The air pressure does not change at the higher temperature. b. The air pressure is higher at the higher temperature. c. The air pressure increases at first but then decreases again as the temperature approaches its higher value. d. The air pressure is lower at the higher temperature. e. The air pressure decreases at first but then increases again as the temperature approaches its lower value. Feedback The correct answer is: The air pressure is higher at the higher temperature. Question 48 Not answered Marked out of 1.00 Not flaggedFlag question Question text On a hot summer day, you turn the thermostat in your house way down. Assume that your house is well sealed and that no air enters or leaves the house. When the air temperature in your house falls a short while later, what statement is correct regarding the air pressure? Select one: a. The air pressure increases at first but then decreases again as the temperature approaches its higher value. b. The air pressure does not change at the lower temperature. c. The air pressure is lower at the lower temperature. d. The air pressure decreases at first but then increases again as the temperature approaches its lower value. e. The air pressure is higher at the lower temperature. Feedback The correct answer is: The air pressure is lower at the lower temperature. Question 49 Not answered Marked out of 1.00 Not flaggedFlag question Question text The air in a room is composed principally of nitrogen, oxygen, and carbon dioxide. These have molecular weights of approximately 28, 32, and 44 g/mol respectively. If the three gases coexist in thermal equilibrium, Select one: a. the kinetic energies, in increasing order are nitrogen, oxygen, and carbon. b. the kinetic energies, in increasing order are carbon, oxygen, and nitrogen. c. the rms speeds, in increasing order are nitrogen, oxygen, and carbon. d. the rms speeds of their molecules are equal. e. the average kinetic energies of their molecules are equal. Feedback The correct answer is: the average kinetic energies of their molecules are equal. Question 50 Not answered Marked out of 1.00 Not flaggedFlag question Question text The number of molecules in one mole of a substance Select one: a. depends on the molecular weight of the substance. b. depends on the density of the substance. c. depends on the atomic weight of the substance. d. is the same for all substances. Feedback The correct answer is: is the same for all substances. Question 51 Not answered Marked out of 1.00 Not flaggedFlag question Question text The temperature at which molecules in a substance have the lowest amount of energy possible is: Select one: a. 0ºC b. 273ºC c. -300ºF d. 0 K Feedback The correct answer is: 0 K Question 52 Not answered Marked out of 1.00 Not flaggedFlag question Question text The temperature of an ideal gas increases from 2°C to 4°C while remaining at constant pressure. What happens to the volume of the gas? Select one: a. It decreases to one-half its original volume. b. It increases slightly. c. It decreases slightly. d. It increases to twice as much. Feedback The correct answer is: It increases slightly. Question 53 Not answered Marked out of 1.00 Not flaggedFlag question Question text To which of the following is high temperature related? Select one: a. high particle kinetic energy b. low particle kinetic energy c. large volume d. zero net energy transfer Feedback The correct answer is: high particle kinetic energy Question 54 Not answered Marked out of 1.00 Not flaggedFlag question Question text When a closed tin is heated, the pressure inside it increases. This happens because: Select one: a. The molecules become heavier and strike the walls of the tin harder. b. The molecules now collide more with the walls of the tin than amongst themselves. c. The molecules move faster and each molecule strikes the walls of the tin more often. d. The number of molecules increases on heating. Feedback The correct answer is: The molecules move faster and each molecule strikes the walls of the tin more often. Question 55 Not answered Marked out of 1.00 Not flaggedFlag question Question text When both the pressure and the absolute temperature of an ideal gas are increased each by a factor of two, the volume occupied by the gas will Select one: a. decrease by a factor of two. b. increase by a factor of two. c. not change. d. increase by a factor of four. e. decrease by a factor of four. Feedback The correct answer is: not change. Question 56 Not answered Marked out of 1.00 Not flaggedFlag question Question text When the translational speed of molecules in an ideal gas triples, by what factor does its absolute temperature change? Select one: a. 9 b. mc025-2.jpg c. mc025-1.jpg d. 3 e. mc025-3.jpg Feedback The correct answer is: 9 Question 57 Not answered Marked out of 1.00 Not flaggedFlag question Question text Which of the following is proportional to the kinetic energy of atoms and molecules? Select one: a. elastic energy b. potential energy c. thermal equilibrium d. temperature Feedback The correct answer is: temperature Question 58 Not answered Marked out of 1.00 Not flaggedFlag question Question text Which of the following statements is NOT a correct assumption of the classical model of an ideal gas? Select one: a. The molecules are in random motion. b. The molecules obey Newton's laws of motion. c. The volume of the molecules is negligible compared with the volume occupied by the gas. d. The only appreciable forces on the molecules are those that occur during collisions. e. The collisions between molecules are inelastic. Feedback The correct answer is: The collisions between molecules are inelastic. Question 59 Not answered Marked out of 1.00 Not flaggedFlag question Question text Air cools as it escapes from a diver's compressed air tank. What kind of process is this? Select one: a. isobaric b. isovolumetric c. adiabatic d. isothermal Feedback The correct answer is: adiabatic Question 60 Not answered Marked out of 1.00 Not flaggedFlag question Question text An adiabatic process is characterized by the absence of Select one: a. heat exchange. b. temperature change. c. entropy. d. None of the above choices are true. e. pressure change. Feedback The correct answer is: heat exchange. Question 61 Not answered Marked out of 1.00 Not flaggedFlag question Question text An ideal gas system is maintained at a constant volume of 4 L. If the pressure is constant, how much work is done by the system? Select one: a. 30 J b. 0 J c. 8 J d. 5 J Feedback The correct answer is: 0 J Question 62 Not answered Marked out of 1.00 Not flaggedFlag question Question text An ideal gas system undergoes an adiabatic process in which it expands and does 20 J of work on its environment. What is the change in the system's internal energy? Select one: a. 20 J b. 0 J c. -5 J d. -20 J Feedback The correct answer is: -20 J Question 63 Not answered Marked out of 1.00 Not flaggedFlag question Question text An ideal gas system undergoes an adiabatic process in which it expands and does 20 J of work on its environment. How much energy is transferred to the system as heat? Select one: a. 5 J b. 20 J c. 0 J d. -20 J Feedback The correct answer is: 0 J Question 64 Not answered Marked out of 1.00 Not flaggedFlag question Question text An ideal gas system undergoes an isovolumetric process in which 20 J of energy is added as heat to the gas. What is the change in the system's internal energy? Select one: a. 0 J b. 5 J c. -20 J d. 20 J Feedback The correct answer is: 20 J Question 65 Not answered Marked out of 1.00 Not flaggedFlag question Question text During an adiabatic compression of an ideal gas Select one: a. no heat is supplied to or removed from the gas. b. the temperature of the gas does not change. c. None of the above choices are true. d. the internal energy of the gas remains constant. e. no work is done on the gas. Feedback The correct answer is: no heat is supplied to or removed from the gas. Question 66 Not answered Marked out of 1.00 Not flaggedFlag question Question text During an isovolumetric process, which of the following does not change? Select one: a. volume b. temperature c. internal energy d. pressure Feedback The correct answer is: volume Question 67 Not answered Marked out of 1.00 Not flaggedFlag question Question text For an ideal gas, the internal energy U depends only on Select one: a. the amount of heat added to it. b. pressure. c. entropy. d. temperature. e. volume. Feedback The correct answer is: temperature. Question 68 Not answered Marked out of 1.00 Not flaggedFlag question Question text How is conservation of internal energy expressed for a system during an adiabatic process? Select one: a. Q = 0, so ΔU = -W b. ΔV = 0, so PΔV = 0 and W = 0; therefore, ΔU = Q c. DT = 0, so ΔU = 0; therefore, ΔU = Q - W = 0, or Q = W d. Q = W = 0, so ΔU = 0 and Umc011-1.jpg = Umc011-2.jpg Feedback The correct answer is: Q = 0, so ΔU = -W Question 69 Not answered Marked out of 1.00 Not flaggedFlag question Question text How is conservation of internal energy expressed for a system during an isovolumetric process? Select one: a. ΔV = 0, so PΔV = 0 and W = 0; therefore, ΔU = Q b. Q = 0, so ΔU = -W c. ΔT = 0, so ΔU = 0; therefore, ΔU = Q - W = 0, or Q = W d. Q = W = 0, so ΔU = 0 and Umc012-1.jpg = Umc012-2.jpg Feedback The correct answer is: ΔV = 0, so PΔV = 0 and W = 0; therefore, ΔU = Q Question 70 Not answered Marked out of 1.00 Not flaggedFlag question Question text How is conservation of internal energy expressed for a system during an isothermal process? Select one: a. ΔV = 0, so PΔV = 0 and W = 0; therefore, ΔU = Q b. Q = 0, so ΔU = -W c. Q = W = 0, so ΔU = 0 and Umc013-1.jpg = Umc013-2.jpg d. ΔT = 0, so ΔU = 0; therefore, ΔU = Q - W = 0, or Q = W Feedback The correct answer is: ΔT = 0, so ΔU = 0; therefore, ΔU = Q - W = 0, or Q = W Question 71 Not answered Marked out of 1.00 Not flaggedFlag question Question text How is conservation of internal energy expressed for an isolated system? Select one: a. Q = 0, so ΔU = -W b. ΔT = 0, so ΔU = 0; therefore, ΔU = Q - W = 0, or Q = W c. Q = W = 0, so ΔU = 0 and Umc014-1.jpg = Umc014-2.jpg d. ΔV = 0, so PΔV = 0 and W = 0; therefore, ΔU = Q Feedback The correct answer is: Q = W = 0, so ΔU = 0 and Umc014-1.jpg = Umc014-2.jpg Question 72 Not answered Marked out of 1.00 Not flaggedFlag question Question text In an isovolumetric process for an ideal gas, the system's change in the energy as heat is equivalent to a change in which of the following? Select one: a. temperature b. volume c. pressure d. internal energy Feedback The correct answer is: internal energy Question 73 Not answered Marked out of 1.00 Not flaggedFlag question Question text In the figure below is a cyclic process carried out in a gas. Which of the following is an accurate statement? mc026-1.jpg Select one: a. The work done in the process is equal to the area enclosed by the cyclic process. b. The work done in the process is equal to the area under the curve adc. c. The work done in the process is equal to the area under ab minus the area under dc. d. The work done in the process is zero. e. The work done in the process is equal under the curve abc. Feedback The correct answer is: The work done in the process is equal to the area enclosed by the cyclic process. Question 74 Not answered Marked out of 1.00 Not flaggedFlag question Question text In the figure below, a gas initially at P1, V1 is caused to change its volume and pressure reversibly such that it moves along the path sketched here. In one cycle the net work done by the gas is thus mc027-1.jpg Select one: a. P2 (V2-V1) b. (P2-P1)(V2-V1) c. P1V1 d. V2 (P2-P1) e. P2V2-P1V1 Feedback The correct answer is: (P2-P1)(V2-V1) Question 75 Not answered Marked out of 1.00 Not flaggedFlag question Question text Suppose you put a closed, sealed can of air on a hot stove burner. The contained air will undergo an increase in Select one: a. pressure. b. internal energy, temperature and pressure. c. internal energy. d. temperature. e. temperature and pressure. Feedback The correct answer is: internal energy, temperature and pressure. Question 76 Not answered Marked out of 1.00 Not flaggedFlag question Question text The first law of thermodynamics is a restatement of the Select one: a. law of heat addition. b. none of these c. conservation of energy. d. Carnot cycle. e. principle of entropy. Feedback The correct answer is: conservation of energy. Question 77 Not answered Marked out of 1.00 Not flaggedFlag question Question text What accounts for an increase in the temperature of a gas that is kept at constant volume? Select one: a. Energy has been removed as work done by the gas. b. Energy has been added as work done on the gas. c. Energy has been added as heat to the gas. d. Energy has been removed as heat from the gas. Feedback The correct answer is: Energy has been added as heat to the gas. Question 78 Not answered Marked out of 1.00 Not flaggedFlag question Question text What thermodynamic process for an ideal gas system has an unchanging internal energy and a heat intake that corresponds to the value of the work done by the system? Select one: a. isovolumetric b. isothermal c. adiabatic d. isobaric Feedback The correct answer is: isothermal Question 79 Not answered Marked out of 1.00 Not flaggedFlag question Question text When an ideal gas does positive work on its surroundings, which of the gas's quantities increases? Select one: a. pressure b. internal energy c. volume d. temperature Feedback The correct answer is: volume Question 80 Not answered Marked out of 1.00 Not flaggedFlag question Question text When an ideal gas is subjected to an adiabatic process Select one: a. the temperature of the gas does not change. b. no work is done on the gas. c. the internal energy of the gas does not change. d. Choices A, B, and C are all true. e. None of the above choices are true. Feedback The correct answer is: None of the above choices are true. Question 81 Not answered Marked out of 1.00 Not flaggedFlag question Question text When mechanical work is done on a system, there can be an increase in Select one: a. its internal energy. b. neither temperature nor internal energy. c. both temperature and internal energy. d. its temperature. Feedback The correct answer is: both temperature and internal energy. Question 82 Not answered Marked out of 1.00 Not flaggedFlag question Question text Which equation describes the net work done for a complete cycle of a heat engine? Select one: a. Wmc019-2.jpg = Qmc019-3.jpg - Qmc019-4.jpg b. Wmc019-8.jpg = PΔV c. Wmc019-1.jpg = Q - ΔU d. Wmc019-5.jpg = Qmc019-6.jpg - Qmc019-7.jpg Feedback The correct answer is: Wmc019-2.jpg = Qmc019-3.jpg - Qmc019-4.jpg Question 83 Not answered Marked out of 1.00 Not flaggedFlag question Question text Which of the following is a thermodynamic process in which a system returns to the same conditions under which it started? Select one: a. an isothermal process b. an adiabatic process c. an isovolumetric process d. a cyclic process Feedback The correct answer is: a cyclic process Question 84 Not answered Marked out of 1.00 Not flaggedFlag question Question text Which thermodynamic process takes place at a constant temperature so that the internal energy of a system remains unchanged? Select one: a. adiabatic b. isovolumetric c. isothermal d. isobaric Feedback The correct answer is: isothermal Question 85 Not answered Marked out of 1.00 Not flaggedFlag question Question text Which thermodynamic process takes place at constant volume so that no work is done on or by the system? Select one: a. isothermal b. adiabatic c. isovolumetric d. isobaric Feedback The correct answer is: isovolumetric Question 86 Not answered Marked out of 1.00 Not flaggedFlag question Question text Which thermodynamic process takes place when work is done on or by the system but no energy is transferred to or from the system as heat? Select one: a. isobaric b. adiabatic c. isovolumetric d. isothermal Feedback The correct answer is: adiabatic Question 87 Not answered Marked out of 1.00 Not flaggedFlag question Question text A chunk of ice with a mass of 1 kg at 0°C melts and absorbs 3.33 ´ 10mc009-1.jpg J of heat in the process. Which best describes what happened to this system? Select one: a. Its entropy decreased. b. Work was converted to energy. c. Its entropy remained constant. d. Its entropy increased. Feedback The correct answer is: Its entropy increased. Question 88 Not answered Marked out of 1.00 Not flaggedFlag question Question text A heat engine has taken in energy as heat and used a portion of it to do work. What must happen next for the engine to complete the cycle and return to its initial conditions? Select one: a. It must do work to transfer the remaining energy as heat to a higher temperature. b. It must give up energy as heat to a higher temperature so work can be done on it. c. It must give up energy as heat to a lower temperature so work can be done on it. d. It must do work to transfer the remaining energy as heat to a lower temperature. Feedback The correct answer is: It must give up energy as heat to a lower temperature so work can be done on it. Question 89 Not answered Marked out of 1.00 Not flaggedFlag question Question text A heat engine would have 100 percent efficiency if its input reservoir were Select one: a. at any finite temperature regardless of the heat sink temperature. b. 100 times hotter than the exhaust sink. c. 100 times cooler than the exhaust sink. d. 1000 times hotter than the exhaust sink. e. any finite temperature if the exhaust sink were at absolute zero. Feedback The correct answer is: any finite temperature if the exhaust sink were at absolute zero. Question 90 Not answered Marked out of 1.00 Not flaggedFlag question Question text According to the second law of thermodynamics, which of the following statements about a heat engine operating in a complete cycle must be true? Select one: a. Heat from a high-temperature reservoir equals the entropy increase. b. Heat from a high-temperature reservoir cannot be completely converted to work. c. Heat from a high-temperature reservoir must be completely converted to work. d. Heat from a high-temperature reservoir must be completely converted to internal energy. Feedback The correct answer is: Heat from a high-temperature reservoir cannot be completely converted to work. Question 91 Not answered Marked out of 1.00 Not flaggedFlag question Question text An electrical power plant manages to transfer 88 percent of the heat produced in the burning of fossil fuel to convert water to steam. Of the heat carried by the steam, 40 percent is converted to the mechanical energy of the spinning turbine. Which best describes the overall efficiency of the heat-to-work conversion in the plant? Select one: a. less than 40 percent b. greater than 88 percent c. 40 percent d. 88 percent Feedback The correct answer is: less than 40 percent Question 92 Not answered Marked out of 1.00 Not flaggedFlag question Question text An ideal heat engine has an efficiency of 50 percent. Which of the following statements is not true? Select one: a. The amount of energy exhausted as heat equals the energy added to the engine as heat. b. The amount of energy exhausted as heat is half the energy added to the engine as heat. c. The amount of energy exhausted as heat equals the work done. d. The amount of work done is half the energy added to the engine as heat. Feedback The correct answer is: The amount of energy exhausted as heat equals the energy added to the engine as heat. Question 93 Not answered Marked out of 1.00 Not flaggedFlag question Question text An important feature of the Carnot cycle is that Select one: a. its efficiency can be 100%. b. its efficiency is determined by the temperatures of the hot and cold reservoirs between which it works and by the properties of the working substance used, and on nothing else. c. no engine can be more efficient than a Carnot engine operating between the same two temperatures. d. its efficiency depends only on the absolute temperature of the hot reservoir used. e. it is an example of an irreversible process that can be analyzed exactly without approximations. Feedback The correct answer is: no engine can be more efficient than a Carnot engine operating between the same two temperatures. Question 94 Not answered Marked out of 1.00 Not flaggedFlag question Question text Entropy is closely related to the Select one: a. both of these b. 1st law of thermodynamics. c. 2nd law of thermodynamics. d. neither of these Feedback The correct answer is: 2nd law of thermodynamics. Question 95 Not answered Marked out of 1.00 Not flaggedFlag question Question text How does a real heat engine differ from an ideal cyclic heat engine? Select one: a. A real heat engine is not isolated, so matter enters and leaves the engine. b. A real heat engine is not cyclic. c. An ideal heat engine converts all energy from heat to work. d. An ideal heat engine is not isolated, so matter enters and leaves the engine. Feedback The correct answer is: A real heat engine is not isolated, so matter enters and leaves the engine. Question 96 Not answered Marked out of 1.00 Not flaggedFlag question Question text Imagine you could observe the individual atoms that make up a piece of matter and that you observe the motion of the atoms becoming more orderly. What can you assume about the system? Select one: a. It is gaining thermal energy. b. Its entropy is decreasing. c. Positive work is being done on the system. d. Its entropy is increasing. Feedback The correct answer is: Its entropy is decreasing. Question 97 Not answered Marked out of 1.00 Not flaggedFlag question Question text More efficient gasoline engines can be built if Select one: a. a better antifreeze is developed. b. None of these - they are already perfectly efficient. c. engines are made to rotate faster. d. materials are developed that can withstand higher temperatures. e. materials are developed that can withstand lower temperatures. Feedback The correct answer is: materials are developed that can withstand higher temperatures. Question 98 Not answered Marked out of 1.00 Not flaggedFlag question Question text The first law of thermodynamics is a restatement of the Select one: a. Carnot cycle. b. principle of entropy. c. conservation of energy. d. law of heat addition. e. none of these Feedback The correct answer is: conservation of energy. Question 99 Not answered Marked out of 1.00 Not flaggedFlag question Question text The ideal efficiency for a heat engine operating between temperatures of 2700 K and 300 K is Select one: a. none of these b. 24%. c. 89%. d. 10%. e. 80%. Feedback The correct answer is: 89%. Question 100 Not answered Marked out of 1.00 Not flaggedFlag question Question text The ideal efficiency for a heat engine operating between the temperatures of 227 degrees C and 27 degrees C is Select one: a. 25%. b. 40%. c. none of these d. 20%. e. 88%. Feedback The correct answer is: 40%. Question 101 Not answered Marked out of 1.00 Not flaggedFlag question Question text The requirement that a heat engine must give up some energy at a lower temperature in order to do work corresponds to which law of thermodynamics? Select one: a. third b. No law of thermodynamics applies. c. first d. second Feedback The correct answer is: second Question 102 Not answered Marked out of 1.00 Not flaggedFlag question Question text Two identical blocks of iron, one at 10 degrees C and the other at 20 degrees C, are put in contact. Suppose the cooler block cools to 5 degrees C and the warmer block warms to 25 degrees C. This would violate the Select one: a. neither of these b. 2nd law of thermodynamics. c. both of these d. 1st law of thermodynamics. Feedback The correct answer is: 2nd law of thermodynamics. Question 103 Not answered Marked out of 1.00 Not flaggedFlag question Question text What occurs when a system's disorder is increased? Select one: a. No work is done. b. No energy is available to do work. c. Less energy is available to do work. d. More energy is available to do work. Feedback The correct answer is: Less energy is available to do work. Question 104 Not answered Marked out of 1.00 Not flaggedFlag question Question text Which of the following is a TRUE statement? Select one: a. It is not possible to convert work entirely into heat. b. All of these statements are false. c. The second law of thermodynamics is a consequence of the first law of thermodynamics. d. It is impossible to transfer heat from a cooler to a hotter body. e. It is possible for heat to flow spontaneously from a hot body to a cold one or from a cold one to a hot one, depending on whether or not the process is reversible or irreversible. Feedback The correct answer is: All of these statements are false.

Thermal


Kaugnay na mga set ng pag-aaral

Chapter 8 Nursing Care of Women with Complications During Labor and Birth

View Set

ch. 6,7,8 (bones physiology & bones hwk)

View Set

Cross-Sectional Anatomy Chest/Abdomen/Pelvis

View Set

Chapter 25 - Suicide (Psych) EAQ's

View Set

Chapter 35 Comfort and Pain Management

View Set